Northern Mountaneous Region mcq pdf download

Northern Mountaneous Region mcq pdf download

???? The Himalayas are formed of parallel fold ranges of which the oldest range is
(A) The Shivalik Range
(B) The Lesser Himalayas
(C) The Great Himalayan Range
(D) The Dhauladhar Range
Ans: (C)


????The foothills region of Himalayas is
(A) Trans – Himalayas
(B) Shivalik
(C) Greater Himalayas
(D) Aravali
Ans: (B)


???? The flat plains along the sub- Himalayan region in North India, are called
(A) Tarai (B) Doon
(C) Khadar (D) Bhabar
Ans: (D)


????Himachal stands for
(A) The Great Himalaya
(B) The Middle Himalaya
(C) Shivalik
(D) Trans-Himalaya
Ans: (B)


????Shivalik Hills are part of which of the following?
(A) Aravali
(B) Western Ghats
(C) Himalaya
(D) Satpura
Ans: (C)


???? Shivalik series was formed in
(A) Eozoic (B) Paleozoic
(C) Mesozoic (D) Cenozoic
Ans: (D)


????. What is the height of Great Himalaya?
(A) 8850 m above sea level
(B) 8815 m above sea level
(C) 8890 m above sea level
(D) 8860 m above sea level
Ans: (A)


????The South of ‘Shivalik’ rock series, Bhabar region is an example of
(A) Midland situation
(B) Intermountain situation
(C) Piedmont situation
(D) Littoral region
Ans: (C)


???? The altitude of Shivalik peaks fall in between
(A) 850-1200 m (B) 750-1100 m
(C) 750-1500 m (D) 750-1300 m
Ans: (A)


???? Main resources of Western Himalayan resource region, are
(A) Forests
(B) Metallic minerals
(C) Carbonic minerals
(D) Atomic minerals
Ans: (A)


????The Himalayan Mountain Ranges are not a part of which of the following states?
(A) Uttarakhand
(B) Uttar Pradesh
(C) Sikkim
(D) Himachal Pradesh
Ans: (B)


????The Lesser Himalaya is located between
(A) Trans Himalaya and Great Himalaya
(B) Shivalik and Great Himalaya
(C) Trans Himalaya and Shivalik
(D) Shivalik and Outer Himalaya
Ans: (B)


???? Which of the following is a part of Aksai Chin region?
(A) Karakoram Range
(B) Shivalik Range
(C) Kashmir Valley
(D) Ladakh Plateau
Ans: (D)


???? Which one of the following is the youngest mountain chain?
(A) Vindhyas (B) Aravalli
(C) Shivaliks (D) Anaimalai
Ans: (C)


???? Which of the following is the correct order of the Himalayan Ranges from South to North in the Western section?
(A) Great Himalaya – Lesser Himalaya – Shivalik
(B) Shivalik – Lesser Himalaya – Great Himalaya
(C) Lesser Himalaya – Great Himalaya – Shivalik
(D) Shivalik – Great Himalaya – Lesser Himalaya
Ans: (B)


???? Between which two mountain ranges is the Valley of Kashmir situated?
(A) Ladakh and Zanskar
(B) Great Himalayas and Pir Panjal
(C) Great Himalayas and Zanskar
(D) Karakoram and Ladakh
Ans: (B)


????Which of the following rock systems in India is the latest one?
(A) Vindhyan (B) Cuddapah
(C) Dharwar (D) Gondwana
Ans: (D)


???? Which one of the following groups is the correct sequence of mountain peaks from East to West?
(A) Everest, Kanchenjunga, Annapurna, Dhaulagiri
(B) Kanchenjunga, Everest, Annapurna, Dhaulagiri
(C) Kanchenjunga, Dhaulagiri, Annapurna, Everest
(D) Everest, Kanchenjunga, Dhaulagiri, Annapurna
Ans: (B)


???? Which one of the following is the correct Northward sequence of the relief features?
(A) Zanskar Ranges, Pir Panjal Ranges, Ladakh Ranges, Karakoram Ranges
(B) Pir Panjal Ranges, Zanskar Ranges, Ladakh Ranges, Karakoram Ranges
(C) Karakoram Ranges, Ladakh Ranges, Zanskar Ranges, Pir Panjal Ranges
(D) Pir Panjal Ranges, Ladakh Ranges, Zanskar Ranges, Karakoram Ranges
Ans: (B)


???? Pir Panjal Range is located/found in
(A) Arunachal Pradesh
(B) Jammu and Kashmir
(C) Punjab
(D) Uttarakhand
Ans: (B)


????In comparison to Eastern Himalaya the value of height of tree-line in Western area is
(A) More
(B) Less
(C) Same
(D) Unrelated variable
Ans: (B)


???? Which one of the following states does not lie along Patkai hills?
(A) Nagaland
(B) Tripura
(C) Manipur
(D) Mizoram
Ans: (B)


????Which mountain range amongst the following is latest in origin?
(A) Ajanta range
(B) Palkonda range
(C) Kaimur Hills
(D) Patkoi ranges
Ans: (D)


????Consider the following statements and select the correct answer from the code given below. Assertion (A): All rivers originating from the Himalayas are perennial. Reason (R): Himalayas receive much of their precipitation from South-Western monsoon. Code
(A) Both (A) and (R) are true and
(R) is the correct explanation of
(A)
(B) Both (A) and (R) are true, but (R) is not the correct explanation of (A)
(C) (A) is true, but (R) is false
(D) (A) is false, but (R) is true
Ans: (B)


???? Identify the correct West to East sequence of the following Hilly Castes.
(A) Khasi-Garo-Naga-Jaintia
(B) Naga-Jaintia-Khasi-Garo
(C) Garo-Khasi-Jaintia-Naga
(D) Jaintia-Naga-Garo-Khasi
Ans: (C)


???? When you travel in Himalayas, you will see the following.
1. Deep gorges
2. U-turn river courses
3. Parallel Mountain ranges
4. Steep gradients causing landsliding Which of the above can be said to be the evidences for Himalayas being young fold mountains?
(A) 1 and 2
(B) 1, 2 and 4
(C) 3 and 4
(D) 1, 2, 3 and 4
Ans: (D)


???? Consider the following statements and select the correct answer from the code given below. Assertion (A): All rivers originating from the Himalayas are perennial. Reason (R): Source of Himalayan rivers is located in glaciers. Code
(A) Both (A) and (R) are true and
(R) is the correct explanation of
(A)
(B) Both (A) and (R) are true, but (R) is not the correct explanation of (A)
(C) (A) is true, but (R) is false
(D) (A) is false, but (R) is true
Ans: (A)


???? Consider the following relief features.
1. Zanskar Range
2. Dhauladhar Range
3. Ladakh Range
4. Karakoram Range The correct South to Northward sequence of the above relief features is
(A) 2, 1, 3, 4 (B) 2, 3, 4, 1
(C) 4, 3, 2, 1 (D) 4, 2, 1, 3
Ans: (A)


???? Examine the map of Jammu and Kashmir given below. 1 2 3 4 The mountain ranges marked 1, 2, 3 and 4, respectively.
(A) Ladakh, Zanskar, Karakoram and Pir Panjal
(B) Karakoram, Pir Panjal, Zanskar and Ladakh
(C) Karakoram, Ladakh, Zanskar and Pir Panjal
(D) Ladakh, Pir Panjal, Karakoram and Zanskar
Ans: (C)


???? Consider the following statements.
1. In India, the Himalayas are spread over five States only.
2. Western Ghats are spread over five States only.
3. Pulicat Lake is spread over two States only. Which of the statements given above is/are correct?
(A) Only 1 and 2 (B) Only 3
(C) Only 2 and 3 (D) Only 1 and 3
Ans: (B)


???? In Himalayan Ranges the types of vegetation changes with altitude due to the following reasons.
1. Decrease in temperature
2. Changes in rainfall
3. Unfertile soil
4. Strong winds Select the correct answer.
(A) 1, 2 and 3 (B) 2, 3 and 4
(C) 1, 3 and 4 (D) 1, 2 and 4
Ans: (A)

Leave a Reply